题目描述

小 Y 是一个爱好旅行的 OIer。一天,她来到了一个新的城市。由于不熟悉那里的交通系统,她选择了坐地铁。

她发现每条地铁线路可以看成平面上的一条曲线,不同线路的交点处一定会设有

换乘站 。通过调查得知,没有线路是环线,也没有线路与自身相交。任意两条不同的线路只会在若干个点上相交,没有重合的部分,且没有三线共点的情况。即,如图所示的情况都是不存在的:aaarticlea/png;base64,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" alt="" />

小 Y 坐着地铁 0 号线,路上依次经过了 n 个换乘站。她记下了每个换乘站可以换乘的线路编号,发现每条线路与她所乘坐的线路最多只有 2 个换乘站。现在小 Y 想知道,除掉她经过的换乘站以外,这个城市里最少有几个换乘站。只有你告诉她正确的答案,她才会答应下次带你去玩呢。

输入输出格式

输入格式:

从文件 metro.in 中读入数据。1

请.注.意.本.题.有.多.组.输.入.数.据。

输入数据的第一行是一个整数 T,表示输入数据的组数。接下来依次给出每组数据。

对于每组数据,第一行是一个整数 n,表示小 Y 经过的换乘站的数目。第二行为 n个用空格隔开的整数,依次表示每个换乘站的可以换乘的线路编号。这些编号都在 1 ~n 之内。

输出格式:

输出到文件 metro.out 中。

对于每组输入数据,输出一行一个整数,表示除掉这 n 个换乘站之外,最少有几个换乘站。

题意:算比较模糊的了,耿直一点就是一条线上一次出现一些点是中转站,会连接一些地铁线路(最多出现两次),如果一个点出现了两次必须要被连起来,求所有连线最小的交点。

题解:

     ①竟然是个爆搜!

     ②首先数据很小,所有的连线方式只有三种,用左端点出线的方向和右端点入线的方向命名就只有:上上,下下,上下,下上(如果你仔细分析一下会发现这样的定义我们已经省略了两种等价的连线)

③按左端点从左到右dfs,维护up和dn两颗BIT分别表示上下的已经连接了的点的右端点,可以统计出相交的情况(需要分类),加一个now>ans的剪枝

     ④然后带着一点点玄学色彩,过了。。。。。  

 #include<cstdio>
#include<iostream>
#include<algorithm>
using namespace std;
const int N=,inf = 1e9;
int n,l[N],r[N],tot;
struct data{
int l,r;
bool operator <(const data&a)const{return l<a.l;}
}A[N];
struct BIT{
int c[N];
void add(int x,int d){for(;x<=n;x+=x&-x) c[x]+=d;}
int ask(int x){int t=;for(;x;x-=x&-x)t+=c[x];return t;}
}Up,Dn;
char gc(){
static char *p1,*p2,s[];
if(p1==p2) p2=(p1=s)+fread(s,,,stdin);
return(p1==p2)?EOF:*p1++;
}
int rd(){
int x=; char c=gc();
while(c<''||c>'') c=gc();
while(c>=''&&c<='') x=x*+c-'',c=gc();
return x;
}
int tmp,ans;
void dfs(int cur){
if(cur>tot||tmp>ans) {ans=min(ans,tmp);return;}
int pre = tmp;
tmp+=min(Up.ask(A[cur].r-) - Up.ask(A[cur].l)
,Dn.ask(n)-Dn.ask(A[cur].l)+Up.ask(n)-Up.ask(A[cur].r));
Up.add(A[cur].r,); dfs(cur+);
Up.add(A[cur].r,-); tmp=pre;
tmp+=min(Dn.ask(A[cur].r-) - Dn.ask(A[cur].l)
,Up.ask(n)-Up.ask(A[cur].l)+Dn.ask(n)-Dn.ask(A[cur].r));
Dn.add(A[cur].r,); dfs(cur+);
Dn.add(A[cur].r,-); tmp=pre;
}
int main()
{ freopen("mzoj1119.in","r",stdin);
freopen("mzoj1119.out","w",stdout);
int T=rd();
while(T--){
n=rd();
for(int i=;i<=n;i++) l[i]=r[i]=; ans=inf;tmp=tot=;
for(int i=,x;i<=n;i++)if(!l[x=rd()]) l[x]=i; else r[x]=i;
for(int i=;i<=n;i++)if(l[i]&&r[i]) A[++tot]=(data){l[i],r[i]};
sort(A+,A+tot+);
dfs();
printf("%d\n",ans);
}
return ;
}//by tkys_Austin;

 

【luogu P4005 清华集训2017】小Y和地铁的更多相关文章

  1. [清华集训2017]小 Y 和地铁(神奇思路,搜索,剪枝,树状数组)

    世界上最不缺的就是好题. 首先考虑暴搜.(还有什么题是从这东西推到正解的……) 首先单独一个换乘站明显没用,只用考虑一对对的换乘站. 那么有八种情况:(从题解偷图)         然后大力枚举每个换 ...

  2. 【清华集训】小Y和地铁

    图已挂,前往luogu 题目: 小 $\rm Y$ 是一个爱好旅行的 $\rm OIer$.一天,她来到了一个新的城市.由于不熟悉那里的交通系统,她选择了坐地铁.她发现每条地铁线路可以看成平面上的一条 ...

  3. 清华集训2017D2T1 小 Y 和地铁(metro)

    题目:https://www.luogu.org/problem/show?pid=P4005 题意:一条线段,给定n个点(n<=44)其中每个点可能对应另外一个点.如果一个点有对应点,那么就要 ...

  4. 【luogu P4007 清华集训2017】小Y和恐怖奴隶主

    题目背景 “A fight? Count me in!” 要打架了,算我一个. “Everyone, get in here!” 所有人,都过来! 题目描述 小 Y 是一个喜欢玩游戏的 OIer.一天 ...

  5. Loj #2324. 「清华集训 2017」小 Y 和二叉树

    Loj #2324. 「清华集训 2017」小 Y 和二叉树 小Y是一个心灵手巧的OIer,她有许多二叉树模型. 小Y的二叉树模型中,每个结点都具有一个编号,小Y把她最喜欢的一个二叉树模型挂在了墙上, ...

  6. 【UOJ#340】【清华集训2017】小 Y 和恐怖的奴隶主(矩阵快速幂,动态规划)

    [UOJ#340][清华集训2017]小 Y 和恐怖的奴隶主(矩阵快速幂,动态规划) 题面 UOJ 洛谷 题解 考虑如何暴力\(dp\). 设\(f[i][a][b][c]\)表示当前到了第\(i\) ...

  7. loj #2325. 「清华集训 2017」小Y和恐怖的奴隶主

    #2325. 「清华集训 2017」小Y和恐怖的奴隶主 内存限制:256 MiB时间限制:2000 ms标准输入输出 题目类型:传统评测方式:文本比较   题目描述 "A fight? Co ...

  8. [LOJ#2324]「清华集训 2017」小Y和二叉树

    [LOJ#2324]「清华集训 2017」小Y和二叉树 试题描述 小Y是一个心灵手巧的OIer,她有许多二叉树模型. 小Y的二叉树模型中,每个结点都具有一个编号,小Y把她最喜欢的一个二叉树模型挂在了墙 ...

  9. [LOJ#2323]「清华集训 2017」小Y和地铁

    [LOJ#2323]「清华集训 2017」小Y和地铁 试题描述 小Y是一个爱好旅行的OIer.一天,她来到了一个新的城市.由于不熟悉那里的交通系统,她选择了坐地铁. 她发现每条地铁线路可以看成平面上的 ...

随机推荐

  1. Flask 文件和流

    当我们要往客户端发送大量的数据比较好的方式是使用流,通过流的方式来将响应内容发送给客户端,实现文件的上传功能,以及如何获取上传后的文件. 响应流的生成 Flask响应流的实现原理就是通过Python的 ...

  2. html{font-size:62.5%}

    为什么要使用html,body{font-size:62.5%}? 使用以下代码查看浏览器的初始font-size: <!DOCTYPE html><html><head ...

  3. 使用JavaScript实现一个俄罗斯方块

    清明假期期间,闲的无聊,就做了一个小游戏玩玩,目前游戏逻辑上暂未发现bug,只不过样子稍微丑了一些-.-项目地址:https://github.com/Jiasm/tetris在线Demo:http: ...

  4. Column Addition~DP(脑子抽了,当时没有想到)

    Description A multi-digit column addition is a formula on adding two integers written like this:

  5. Andrew Ng机器学习第一章——初识机器学习

    机器学习的定义 计算机程序从经验E中学习,解决某一任务T.进行某一性能度量P,通过P测定在T上的表现因E而提高. 简而言之:程序通过多次执行之后获得学习经验,利用这些经验可以使得程序的输出结果更为理想 ...

  6. Web Api HttpWebRequest 请求 Api 及 异常处理

    HttpWebRequest request = WebRequest.CreateHttp(url); request.Method = "post"; request.Head ...

  7. Docker的容器操作

    启动一次性运行的容器 入门级例子:从ubuntu:14.04镜像启动一个容器,成功后在容器内部执行/bin/echo 'hello world'命令,如果当前物理机没有该镜像,则执行docker pu ...

  8. Extensions in UWP Community Toolkit - Mouse Cursor

    概述 UWP Community Toolkit Extensions 中有一个为 Mouse 提供的扩展 - Mouse Cursor Extensions,本篇我们结合代码详细讲解 Mouse C ...

  9. 前端之HTML内容

    一.HTML介绍 1.Web服务本质 当我们在浏览器中输入一个url后打开一个页面这个过程实质是一个网络编程中的sockt服务端接受指令并发送指令的一个过程.本质顺序是: 浏览器发请求——>HT ...

  10. python之路1

    python之路 http协议 html HTML2 CSS选择器 CSS属性操作 CSS属性操作/下 JavaScript(js)/上 JavaScript的对象 JavaScript的对象/下 前 ...